1
$\begingroup$

We know following theorem by Schur: Suppose that $f(x) \in \mathbb{Z}[x]$ is a polynomial such that exists an integer $m$ such that $f(n) = m^2$ for every integer $n$. Then $f(x) = g(x)^2$ for some $g(x) \in \mathbb{Z}[x]$.

Is this true if I using $3,4,5 \cdots$ instead of $2$?

$\endgroup$
4
  • $\begingroup$ Should we assume that your $m$ is not fixed? Meaning, is there a single $m \in \mathbb{Z}$ such that $f(n) = m^2$ for all integers $n$? If so, $f(x) := m^2$. $\endgroup$ Oct 28, 2012 at 17:45
  • $\begingroup$ Yes it is true. See Cor. 3.3 in isibang.ac.in/~sury/polys.pdf $\endgroup$
    – J.C. Ottem
    Oct 28, 2012 at 18:19
  • $\begingroup$ (assuming the quantifiers are 'for every $n$ there exists and $m$ such that $f(n)=m^2$).. $\endgroup$
    – J.C. Ottem
    Oct 28, 2012 at 18:23
  • $\begingroup$ The same finite-difference trick that solves the square-root version of this problem (MO#110904) works here too. $\endgroup$ Oct 28, 2012 at 18:33

2 Answers 2

2
$\begingroup$

I think the proposer tried to say that if for every integer $n$ there is an integer $m$ such $f(n)=m^2$, then $f(x)=g(x)^2\dots$.

The generalization from exponent $2$ to higher exponents $e$ is an exercise in the second volume of the classical Polya/Szegö problem book. I don't have it at hand now, so I cannot give a more precise reference.

If one allows more sophisticated tools, then it follows immediately from Hilbert's irreducibility theorem: Let $t$ be another variable, and factorize $f(x)-t^e$ over $\mathbb Q(t)$ into irreducible factors $P_i(t,x)$. By Hilbert, there are infinitely many integers $n$ such that $P_i(n,x)$ is irreducible for all $i$. On the other hand, $f(x)-n^e$ has an integral root for each $n$. Thus there is an index $i$ such that $P_i(t,x)$ has degree $1$ with respect to $x$, so $f(x)-t^e$ has a root in $\mathbb Q(t)$. Gauss' lemma yields that this root is in $\mathbb Q[t]$, so $f(t)=g(t)^e$ for a polynomial $g$ with rational coefficients. One quickly gets that the coefficients of $g$ then are indeed integers.

$\endgroup$
1
$\begingroup$

Yes. See e.g. http://www.isibang.ac.in/~sury/polys.pdf Corollary 3.3.

$\endgroup$

Your Answer

By clicking “Post Your Answer”, you agree to our terms of service and acknowledge you have read our privacy policy.

Not the answer you're looking for? Browse other questions tagged or ask your own question.